LSAT and Law School Admissions Forum

Get expert LSAT preparation and law school admissions advice from PowerScore Test Preparation.

 lunalondon
  • Posts: 20
  • Joined: Mar 26, 2017
|
#35831
Hello Powerscore,

I've actually got another question on this one. Why can't D be correct? If the annual number of visitors has NOT increased steadily, then the minimum wage would not really seem to "adversely affect the museum-going public." Or perhaps it could adversely affect the loyal museum goers? Thanks!
User avatar
 Jonathan Evans
PowerScore Staff
  • PowerScore Staff
  • Posts: 726
  • Joined: Jun 09, 2016
|
#35860
LunaLondon, I am sorry! Nikki was right and I was wrong. I have corrected the explanation accordingly and apologize for my error. Negate the quantity statement here for an accurate negation. Thank you for pointing this out!
User avatar
 Jonathan Evans
PowerScore Staff
  • PowerScore Staff
  • Posts: 726
  • Joined: Jun 09, 2016
|
#35861
Hi, LunaLondon,

To answer your second question, answer choice (D) is not a necessary belief of the author and, if this trend continues, would actually weaken the author's conclusion. Does the author have to believe that the number of annual visitors has increased steadily? Not necessarily. Could it be that the number of annual visitors has NOT increased steadily and that "the museum will be forced either to raise admission fees or to decrease services"?

The conclusion is still a possibility, so this particular answer fails the Assumption Negation TestTM.
 lunalondon
  • Posts: 20
  • Joined: Mar 26, 2017
|
#35875
Thanks!
 Blueballoon5%
  • Posts: 156
  • Joined: Jul 13, 2015
|
#50423
Hello! When tackling answer choice A, I made an error in the Negation Technique. However, I am not sure where I went wrong.

My negation of answer choice A: "Some of the museum's employees are paid significantly more than the minimum wage." (I simply removed the "not").

Thus, with this negation, I eliminated answer choice A because I didn't think it necessarily hurt the argument that some of the employees happened to be paid over the minimum wage. This seems like a "so-what" kind of answer. Whether some employees were previously paid over the minimum wage, it doesn't change the fact that the 5% increase can increase the museum's operating expenses.
 Brook Miscoski
PowerScore Staff
  • PowerScore Staff
  • Posts: 418
  • Joined: Sep 13, 2018
|
#62679
Blueballoon,

Recall my earlier advice (albeit this same evening) that you should negate the choice in a way that disrupts the logical relationship presented in the choice. Let's think about the choice and your negation.

Choice: Some employees are not paid significantly more than minimum wage.
Your negation: Some employees are paid significantly more than minimum wage.

That is not a negation, since both of the statements can be true at the same time. Some are, while some are not. To disrupt the logical relationship presented in the choice, you need a negation that contradicts the answer choice. Let's look at the other option:

Choice: Some employees are not paid significantly more than minimum wage.
Negation: All of the employees are paid significantly more than the minimum wage.

These choices cannot be true simultaneously, so we have a negation. Looking to the effect on the stimulus, if all of the employees already make well over the minimum wage, the law will not have any immediate effect on the museum.
 lsatprep1215
  • Posts: 33
  • Joined: Dec 16, 2019
|
#73592
I chose A as my answer for this question, however, it is not what I pre phrase. My original pre phrase is that there are no other ways to help cover the operating expense except to raise admission fees or to decrease services. If there are other ways, then this conclusion will be weaken/ruined. I did not find what I want to look for so I picked A because I know the negation of this answer will also attack the "will significantly increase the museum’s operating expenses". Although I chose the correct answer, I still want to know is there any problem with my pre phrase?
User avatar
 KelseyWoods
PowerScore Staff
  • PowerScore Staff
  • Posts: 1079
  • Joined: Jun 26, 2013
|
#73619
Hi lsatprep!

You prephrased an assumption of the argument, but you didn't prephrase the answer choice very well. It's a tricky distinction, but remember that most arguments have multiple assumptions. Sometimes you're able to prephrase which specific assumption they're going to give you in the answer choice, but sometimes you prephrase one specific assumption and they give you a totally different assumption in the answer choice (as happened to you here). So make sure that your prephrase is always broad enough to allow you to consider answer choice you may not have specifically anticipated. For instance, in this question, you could have said that there is one specific assumption that you see (the one you prephrased) but that there may be other assumptions necessary for the argument that raising the minimum wage is going to adversely affect the museum-going public.

Ultimately, you approached these answer choices the right way. You didn't see what you prephrased, so you fell back on your Assumption-Negation technique and it led you to the correct answer. It's important to not get so attached to a specific prephrase that you're unable to see answer choices that answer the question even if they're not exactly what you were thinking of. And you did that successfully!

Hope this helps!

Best,
Kelsey
User avatar
 Albertlyu
  • Posts: 98
  • Joined: Jul 18, 2020
|
#77272
In flaw questions, we have to assume the evidence is right and find the gap between evidence and conclusion, but can we question the validity of the evidence in assumption questions? Here is an example that the evidence is inconsistent with the context that the stimulus provides:

Stimulus text removed due to copyright concerns.

the question asks for a necessary assumption, my question is: the stimulus has already stated explicitly that"the mandate will significantly increase the museum's operating expense", but the right answer choice says"some of the employees are not paid more than the minimum wage", which indicates the operational expense won't be increased by the mandate, please can anyone help? thanks
 Rachael Wilkenfeld
PowerScore Staff
  • PowerScore Staff
  • Posts: 1358
  • Joined: Dec 15, 2011
|
#77319
Hi Albert,

We are looking for the answer choice here that is needed in order for the conclusion here to follow. We need something that will tell us their expenses will go up because of the new mandate. Let's think about who the mandate would effect. If it's a minimum wage mandate, it will impact those at or near the minimum wage, as well as those who employ them. Here, we need to know that some employees are at or near minimum wage, or it wouldn't follow that the expense would go up.

Let's look at a simple example. Let's say that the museum pays all employees twice the minimum wage. A five percent increase in minimum wage would have no effect on the museum budget, because they are already paying more than that level However, if the museum was paying everyone minimum wage, it would be a significant change in their budget. That means that we need at least some people making at or near minimum wage in order for our conclusion to follow. Otherwise, if no one is at that level, there wouldn't be any budget change based on the mandate.

Hope that helps!
Rachael

Get the most out of your LSAT Prep Plus subscription.

Analyze and track your performance with our Testing and Analytics Package.